Chapter 4: halls marriage theorem Flashcards

1
Q

EXAMPLE: in a group of 7 men and 6 women

woman 1 knows men 1',2',3'
woman 2 knows men 2',3'
woman 3 knows men3',5',7'
woman 4 knows men1',2'
woman 5 knows men1',2',3'
woman 6 knows men 4',5',6'

can each find a husband from the man she knows?

A

this is impossible

e.g women 1,2,4 and 5 only know 3 Men between them (1’,2’,3’) so they can’t get married to different men they know

relates to LEMMA: for plausibility and full matching

ie iff for each subset of r women they know at least r men between them

How well did you know this?
1
Not at all
2
3
4
5
Perfectly
2
Q

consider a job allocation problem
finite set of jobs and finite set of people

corresponds to counting rook polynomials

A

Finite set j of jobs
Finite set P of people
For each j ∈J, a subset C[j]⊆P of candidates = those qualified for job

corresponds to board with rows labelled J and columns labelled P with (j,p) is unblocked iff p∈C[j]

Given subset U ⊆J: a partial matching on U gives injective map m:U→ P with m(j)∈C[j] for all j

  • m(j)∈C[j] means we assign j to person m(j) who is qualified to do j
  • injective means we do not assign 2 different jobs to the same person

*rook polynomials to see if there is at least a full matching
a full matching is one such that map m with U=all J

How well did you know this?
1
Not at all
2
3
4
5
Perfectly
3
Q

C[U] in job allocation

A

C[U]= ∪_{j∈U} C[j]

e.g. C[ {i,j,l}] = C[j] ∪C[k] ∪C[l]

C[U]={people qualified to do at least one of the jobs in U}

How well did you know this?
1
Not at all
2
3
4
5
Perfectly
4
Q

A full matching for u, set of jobs PLAUSIBILITY definition

A

u is
*implausible if |C[u]| is LESS THAN |u|
barely plausible if |C[u]| = |u|
Very plausible if |C[u]| is MORE THAN |u|

*plausible if |C[u]| ≥ |u|

(so plausible if barely plausible or very plausible)

How well did you know this?
1
Not at all
2
3
4
5
Perfectly
5
Q

LEMMA: for plausibility and full matching

A

if there is a full matching m: U→ P then every subset U ⊆J is plausible

proof: Suppose U={j_1,…,j_r}. Then m(j_i) ∈C[j_i] ⊆C[u] for all i, and m(j_1),..,m(j_r) are all different as m is injective, thus |C[u]| ≥ |u|

How well did you know this?
1
Not at all
2
3
4
5
Perfectly
6
Q

COROLLARY:for plausibility and full matching

A

if there are any impossible/implausible sets, then the problem is not solvable

How well did you know this?
1
Not at all
2
3
4
5
Perfectly
7
Q
EXAMPLE: is the problem solvable
                Pa Qui Ru Ste Tess
\_\_\_\_\_\_\_\_\_\_\_\_\_\_\_\_\_\_\_\_\_
Artist       |x     x     
Baker      |       x   x             x
Courrier  |x     x
Dentist    |x     x   x      x     x
Electr''     |x     x

x means qualified

A

C[{A}]={P,Q} so {A} is very plausible

C[{B,D}]={P,Q,R,S,T} so {A} is very plausible

(candidates able to do at least one of the jobs)

C[{A,C}]={P,Q} so {A,C} is barely plausible

C[{A,C,E}]={P,Q} so {A,C,E} is implausible

thus the problem is not solvable

we only need to find one example such that it’s implausible (try out different jobs)

How well did you know this?
1
Not at all
2
3
4
5
Perfectly
8
Q

halls marriage theorem

A

is every set of jobs is plausible then there exists a full matching

(converse of lemma)

if there is a full matching m: U→ P then every subset U ⊆J is plausible

  • marriage version iff for every set of size r must have at least r men known
  • proof by induction for number of jobs
How well did you know this?
1
Not at all
2
3
4
5
Perfectly
9
Q

NEW ASSIGNMENT FROM PART ASSIGNMENT EXAMPLE

4 jobs and 4 people assigned

j_0 has not been assigned (to a person)
p_0 is qualified for j_0 but has already been assigned to j_1

p_1 is qualified for j_1 but has already been assigned to j_2

p_2 is qualified for j_2 but has already been assigned to j_3

p_3 is qualified for j_3 AND has NOT been given a job

A

by forming open zigzag

we have already
j_0-----------p_0
            /
j_1------------p_1
           /
j_2-----------p_2
          /
j_3-----------p_3

(/ mark arrows of solid line: person is assigned and qualified)

dotted line:person is qualified

new assignment:

j_0________>p_0

j_1________>p_1

j_2________>p_2

j_3________>p_3

How well did you know this?
1
Not at all
2
3
4
5
Perfectly
10
Q

OPEN ZIGZAG

A

a sequence like prev example (j_0,..,j_n; p_0,..,p_n) where p_n has not been assigned a job is called an open zigzag. changing the assignment (as before) is called flipping

if we can find an open zig zag then we can flip it,which modifies and extends m (So It covers j as well as j)

How well did you know this?
1
Not at all
2
3
4
5
Perfectly
11
Q

IS THERE ALWAYS AN OPEN ZIGZAG?

A

if we have a possible job allocation problem, a partial solution
m_1: J_1 → P
and an extra job
j_0 ∈ J\J_1
then we can always find an open zig zag and flip it to get a partial solution
m: J_1∪{j_0} → P

we can use this repeatedly for a full matching

How well did you know this?
1
Not at all
2
3
4
5
Perfectly
12
Q

example:

J={A,B,C,D}
P={p,q,r,s}

C[A]={p,q,r,s}
C[B]={q,r,s}
C[C]={r,s}
C[D]={p}

A

For ABC make A→p, B→q,C→r but then D cannot be assigned without reassignment

p q r s
: \ : \ : \ :
D A B C

we can find open zig zag as plausible C[D] and flip for complete matching

A→q, B→r,C→s, D→p

could use rook polynomial board to visualise but this gave us a required solution

How well did you know this?
1
Not at all
2
3
4
5
Perfectly
13
Q

job allocation with

|P| bigger than |J|

A

some people won’t be assigned a job

*we consider a subset T⊆P of people who are poor and really need to be assigned a job

How well did you know this?
1
Not at all
2
3
4
5
Perfectly
14
Q

THEOREM 54’

addition for plausibility

A

suppose that every subset U⊆J is plausible

ie |C[u]| ≥ |u|.

suppose also that every u has
|C[u]∩T| ≥ |u| + |T| - |J|

then there is a full matching in which every person in T is assigned

(what we are looking for)

proof: by choosing a set not available to T..

How well did you know this?
1
Not at all
2
3
4
5
Perfectly
15
Q

DEF Q[p]

A

in a job allocation problem as before for each person p put

Q[p]={jobs p can do}
={j∈J: p∈C[j] }

How well did you know this?
1
Not at all
2
3
4
5
Perfectly
16
Q

Example: what are C[j]s and Q[p]s?

    p  q  r  s
A [  ][  ][  ][  ]
B [x][  ][  ][  ]
C [x][x][  ][  ]
D [  ][x][x][x]

x means unshaded

A
C[A]={p,q,r,s}
C[B]={q,r,s}
C[C]={r,s}
C[D]={p}
candidates able to do job_A_
Q[p]= {A,D}
Q[q]= {A,B}
Q[r]= {A,B,C}
Q[s]= {A,B,C}

jobs doable by candidate_p_

How well did you know this?
1
Not at all
2
3
4
5
Perfectly
17
Q

PROP 55: full matching

relationship between C[j] and Q[p]

A

suppose there is a constant d bigger than 0 st

1) for all jobs j there are at least d candidates: |C[j]| ≥ d
2) each person p can do at most d jobs: |Q[p]| ≤ d

Then every set of jobs is plausible, so there is a full matching (*by halls thm).

special case if :|C[j]|= |Q[p]| then there is a full matching

18
Q

COROLLARY 56:

full matching

relationship between C[j] and Q[p]

and TALENTED PEOPLE

A

suppose
1)for all j |c[j]| = d
(every job can be done by d candidates)

2)for all p |Q[p]| ≤ d
(every person can do less than or equal to d jobs)

and put t= {p : |Q[p]| =d } ={talented people}.
(talented people are those who can do d jobs)

Then there is a full matching in which all in T get jobs.

by prop 55 we know there is a full matching but the talented people are the ones getting the jobs by thm 54’ addition for plausibility

ie if we have people who can do d jobs, where each job can be done by d candidates, and a full matching is possible then these talented people get jobs

19
Q

TRANSVERSAL VERSION

A

Suppose we have finite sets A_1,…, A_r

A distinct set of Representatives is a list a_1,…,a_r with all a_i’s different and a_i ∈ A_i

for example a family with one representative from each family only

for example A_1 is boys girl 1 knows, etc

ie for U={1,2,4,5} |C[U]| = 3 less than |U| :U isn’t plausible, so no full matching

20
Q

THM:

a TRANSVERSAL

VERSION

distinct set of Representatives if

A

there is a distinct set of Representatives if for any set {i_1,…,i_k} of indices we have

|A_{i_1} ∪…∪A_{i_k} | ≥ k

for any subset of the representatives union is at least k ie all different: iff for sets A_1,..,A_r any collection of r of the sets contain at least r elements

21
Q
EXAMPLE: Sets
A_1={1, 3}
A_2={2, 3}
A_3={1,3,4,5}
A_4={2,4,6}
A_5={1,5}
A_6={1,2}

marked in red 1

A
A_1={_1_, 3}
A_2={2, _3_}
A_3={1,3,_4_,5}
A_4={2,4,_6_}
A_5={1,_5_}
A_6={1,_2_}

marked in red 1

families: curlyA= {A_1,..,A_6}

have distinct representatives
(a transversal)

X={1,3,4,6,5,2} any order

a_1=1, a_2=3,…,a_6=2 gives a complete set of Representatives, each distinct value and not all values need be chosen

22
Q
EXAMPLE: Sets
A_1={1, 2,3}
A_2={2, 3}
A_3={3,5,7}
A_4={1,2}
A_5={1,2,3}
A_6={4,5,6}

marked in red 1

A

don’t have distinct representatives since for example 4 sets:

A_1, A_2, A_4, A_5
contain between them only three members
1,2,3

A_1∪A_2∪A_4∪A_5 ={1,2,3}

and we need the union of any are of those sets to contain at least are elements

Hence family curlyA has no transversal

23
Q

team version

A

suppose we have set J of jobs , set P of people

and for each job J we have a subset C[j]⊆P of candidates.

However, suppose we now need a team of people for each job, size m_j for job j

U⊆J is plausible if and only if

|C[U]| ≥ Σ_{u∈U} m_u

e.g {j_1,j_2,j_3} is plausible iff

number of candidates able to do any ONE of 3 jobs |C[j_1,j_2,j_3]| ≥ m_{j_1}+m_{j_2}+m_{j_3} sum of team sizes for each job

proof from allocation is plausible if and only if every subset U⊆J is plausible

24
Q

tournament definition

A

the tournament of employers consists of n Choose 2 games:
(n)
(2)

  • each player plays each of the others once
  • each game results in a win for one player
25
Q

tournament example
A B C D E play a tournament games and winners shown:

marked red = A*

A

AB AC AD* AE BC BD BE CD CE D*E

or winner→loser

↓ ↓ ↓
A → B → D → E → C
↑↑ ↑

(lines drawn between the players)

scores table:
A:3 B:2 C:2 D:2 E:1

scores seq
3221 always written in descending order

note that knowing A→B→D→E→C helps us draw
this is a winning line

26
Q

DEF winning line

A

winner→loser

↓ ↓ ↓
A → B → D → E → C
↑↑ ↑

(lines drawn between the players curved/cross each other from winner to loser)

a winning line is a sequence p_1,…,p_n containing each player exactly once, such that p_1 beats p_2, p_2 beats p_3,…,p_{n-1} beats p_n.

27
Q

LEMMA: winning line

A

For any tournament of employers, there exists and winning line.

proof by induction. Choose one player p* and use induction hypothesisp_1,..,p_{n-1} to show where p* goes if they beat :
p_1- beginning
no-one, last player
o/w let p_i be the last player in the sequence that beats p* so p* beats p_{i+1}
and
there is a wl
p_1,..,p_i,p*,p_{i+1},…,p_{n-1}

28
Q

LEMMA: nCk used for tournaments

A

For 0≤k≤n we have

n) (k) (n-k
(2) = (2) + k(n-k) + ( 2 )

29
Q

scores of tournaments of n players

consider a list of non negative integers

when can this be a list of scores for a tournament of n players?

FIRST CONDITION:

A

FIRST CONDITION:

each of the
(n)
(2)
games gives a score of 1 to one player and 0 to the other

therefore
s_1+..+s_n= total scores of all players= total number of games=
(n)
(2)

ASSUME this is the case

30
Q

sum of scores of tournament for a subset of players

A

FOR ANY SUBSET U⊆{1,…,n} we put s_u=Σ_{i∈u} s_i

e.g
s_{1,4,7}=s_1+s_4+s_7

we always have s_u + s_{u^c} =
(n)
(2)

31
Q

DEFINITION:
the list s is plausible if

of scores

A

the list s is plausible if and only if for all u we have

s_u ≥
(|u|)
( 2)

from first condition

32
Q

LEMMA:

the list s is plausible if

of scores

with LEMMA: nCk used for tournaments

A

if s is plausible then for all u we have
s_u ≤
(|u|)
( 2) + |u|(n-|u|)

In other words, if |u|=k then
s_u ≤
( k)
( 2) + k(n-k)

*reversible:so plausible if and only if for all u, |u|=k then
s_u ≤
( k)
( 2) + k(n-k)

33
Q

LEMMA:

A

if the numbers s_i are the scores from a tournament, then the list is plausible

ie
s_1+..+s_n= total scores of all players= total scores from games between members of u + total from games between u and u^c =
( k)
( 2) + a number between 0 and (n-k))

so 
( k)
( 2)
≤ s_u ≤
( k)
( 2)   +     k(n-k)
34
Q

THEOREM 68
LANDAU

*reversible

A
if the list s_1,..,s_n is plausible
(s_1+..+s_n= 
(n) 
(2)
)

Then there exists a tournament with scores s_1,..,s_n

35
Q

addendum: THEOREM 68

LANDAU

A

suppose that the numbers s_1,..,s_n are in decreasing order. Then for any u with |u|=k we have

s_{n-k+1} +…+s_n ≤ s_u ≤ s_1+…+s_k

therefore s is plausible if and only if for All K

s_1+…+s_k ≤
( k)
( 2) + k(n-k)

if and only if for All K

s_{n-k+1} +…+s_n ≥
(k)
(2)

(=last k s’s)

36
Q

SUMMARY FOR SCORES OF TOURNAMENTS AND EQUIVALENT STATEMENTS

CONDITIONS

A

For s_1,..,s_n in N with s_1+…s_n =
(n)
(2)
equivalent:

(1) There is a tournament of employers with scores s_1,..,s_n

(2) The sum of any k of the s_i is at least
(k)
(2)

(3) The sum of any k of the s_i is at most
(k)
(2) + k(n-k)

***ordered s_1 ≥… ≥s_n then
LAST k at least kC2
FIRST k at most kC2 + k(n-k)

37
Q

EXAMPLE:
S=532221
are scores of a tournament?

A

n=6
CHECK: s_1+…s_n =15 = 6C2

seems to satisfy conditions:
1  ≥ 1C2=0
2+1 = 3  ≥ 2C2=1
2+2+1=5  ≥ 3C2 = 3
....
5+...+1= 6C2

example of a tournament:

right against top
    1   2   3   4    5  6
1 [X][W][W][W][W][W]
2[ L][X ][ L][W][W][W]
3[ L][W][X][ L ][ L][W]
4[ L][ L][W][ X][ L][W]
5[ L][ L][W][W][ X][ L]
6[ L][ L][ L][ L][W][ X]

x means doesnt play
e.g player 5 lost against player 2

notice symmetric

38
Q

EXAMPLE:
S=4442111
are scores of a tournament?

A

n=7 but
s_1+..s_7= 17 not equal to 21= 7C2

or sum of last four: 1+1+1+2=5 less than 4C2 =6
violates condition

so cannot be a list of scores from a tournament

39
Q

EXAMPLE:
S=5444110
are scores of a tournament?

A

sum of last 3 is
2 which is less than 3C2 and thus not plausible

1,1,0 mean 3 games played between them and thus should share 3 wins

40
Q

EXAMPLE:
S=987654321
are scores of a tournament?

A

9 players n=9

scores for tournament in which they lower numbered player always wins i.e. 1 better than 2, 2 better than 3 etc

41
Q

EXAMPLE:***
For an odd number n=2m+1 we can imagine a tournament in which i beats j if and only if

j-i ∈{1,..,m} mod 2m+1

A

m=3

    1    2   3    4    5  6    7
1 [X][W][W ][W ][ L ][ L ][ L]
2[ L][X ][W ][W ][W][ L ][ L]
3[ L][ L ][X ][W ][W ][W][ L]
4[ L][ L ][ L ][ X ][W][W][W]
5[W][ L ][ L ][ L ][ X][ W][W]
6[W][W][ L ][ L ][ L ][ X][W]
7[W][W][W ][ L ][ L ][ L ][X]

This has scores m,…,m
(repeated n times)

because this is the score sequence from a tournament, it must satisfy the plausibility condition. We can see this more directly:

(sum of any k scores, with k≤n )

=km=
k [(n-1)/2]≥ kC2

42
Q

problem: construct a tournament with scores 7777722222

A

10 players

we can :
construct tournament with players 1-5 with scores 22222 by case m=2 of ***

and 6-10 with scores 22222.

Then consider a combo tournament with players 1,..,10 in which 1-5 always beat 6-10.